Add problems for Serway and Jewett v8's chapter 31.
[course.git] / latex / problems / Serway_and_Jewett_8 / problem31.30.tex
1 \begin{problem*}{31.30}
2 A rectangular coil with resistance $R$ has $N$ turns, each of length
3 $l$ and width $w$ as shown in Figure~P31.30.  The coil moves into a
4 uniform magnetic fiield $\vect{B}$ with constant velocity $\vect{v}$.
5 What are the magnitude and direction of the total magnetic force on
6 the coil \Part{a} as it enters the magnetic field, \Part{b} as it
7 moves within the field, and \Part{c} as it leaves the field?
8 \begin{center}
9 %          x x x Bin
10 %  +--+ v  x x x x
11 % w|  |->  x x x x
12 %  +--+    x x x x
13 %   l      x x x x
14 \begin{asy}
15 import Mechanics;
16 import ElectroMag;
17
18 real w = 5mm;
19 real l = 6mm;
20 real u = 1.5cm;
21
22 Vector B = BField(phi=-90);
23 vector_field((l/2 + 7mm + u/2, 0), width=u, height=u, v=B, outline=dashed);
24 Vector v = Velocity((l/2, 0), "$\vect{v}$");  v.draw();
25 path p = scale(l, w)*shift((-0.5, -0.5))*unitsquare;
26 draw(p);
27 draw(shift(1.5pt*dir(-45))*p);
28 Distance Dl = Distance((-l/2, -w/2), (l/2, -w/2), "$l$", offset=6pt);
29 Dl.draw();
30 Distance Dw = Distance((-l/2, w/2), (-l/2, -w/2), Label("$w$", embed=Shift),
31     offset=6pt);
32 Dw.draw();
33 \end{asy}
34 \end{center}
35 \end{problem*}
36
37 \begin{solution}
38 \end{solution}
39